LSAT and Law School Admissions Forum

Get expert LSAT preparation and law school admissions advice from PowerScore Test Preparation.

 yrivers
  • Posts: 68
  • Joined: Mar 15, 2017
|
#33451
Could someone provide an explanation for June 1991 LR 1 #23? I chose A (Of the readers who received questionnaires, 90 percent returned them.) But the key says it's C.
 Ricky_Hutchens
PowerScore Staff
  • PowerScore Staff
  • Posts: 59
  • Joined: Oct 12, 2015
|
#33462
Let's consider what each of those answers actually accomplishes for the stimulus. We have a journal that has sent a survey out. A certain number of people returned that survey. 62% answered yes.

Answer "A" says that 90% of the people who received the survey returned. This sounds great because 90% is a large participation rate, right? Maybe. Imagine that the potential readership of this journal is 1 million people and that they only sent the survey to 100 people. Now only 90 people out of a possible million have voted on whether the change is any good. There is a very good possibility that those 90 people are not representative of the larger class. Therefore, while Choice A certainly doesn't hurt the argument, it may not help it either.

On the other hand, answer C states outright that the 62% from the survey is representative of the entire potential readership. That means that no matter how big that readership is, 62% will like the new format. C actually eliminates the possibility of an unrepresentative sample, the very problem that A could run into.
 yrivers
  • Posts: 68
  • Joined: Mar 15, 2017
|
#33540
This is helpful. But C states "The percentage of surveyed readers who like the format change was almost the same as the percentage of the entire potential readership who would LIKE FORMAT CHANGE."

For me that last piece that I put in caps indicates a bias among the sample/folks who were surveyed. If all the people who were surveyed wanted a format change, aren't they more likely to like the format change that's been proposed? If C didn't have that piece that I put in caps, I'm 100% on board with the explanation offered. But still a bit confused.

Also option E is really hard to understand. I had to read it several times to understand how it relates to the stimulus. Is that normal or should I be understanding 100% what it means to eliminate?
 Robert Carroll
PowerScore Staff
  • PowerScore Staff
  • Posts: 1787
  • Joined: Dec 06, 2013
|
#33563
yrivers,

Those words do not indicate a bias in the people surveyed. Answer choice (C) indicates that the 62% number would still be true even if the survey were extended to the entire potential readership. Think about it this way. Imagine 100 people returned the survey. 62 of those people liked the format change. Answer choice (C) is saying that if we took every potential reader, 62% of that group would also like the format change. I think your confusion arises from thinking that answer choice (C) said something only about people who like the format change, which would be dealing with a biased group. It's not; it's saying "of all potential readers," which is not a biased group, 62% would like the format change. Thus, if you extended the total number of people, you'd get a similar percent liking the format change, which means it seems like the survey was accurate.

If an answer choice is confusing, keep always in mind what you're looking for in an answer choice. This is a Strengthen question. After reading the question, you'll have a prephrase in mind. If answer choice (E) doesn't match that prephrase, try to interpret how, if at all, answer choice (E) could be strengthening the argument. In this case, it's indicating a possible bias in people's responses to the survey - a larger percentage of dissatisfied than of satisfied people responded. How would that affect the argument? In this case, it wouldn't affect it in any positive way, so it can be discarded.

Robert Carroll
 Lawyered
  • Posts: 23
  • Joined: Jun 13, 2017
|
#36030
I still don't get it

I narrowed it down to two answer choices C & E.

I picked E because it seemed like 99% of the people who didn't like the old format & 50% of the people who liked the old format returned the question.

The conclusion was "the decision was made to introduce the new format".

Wouldn't 99% of the people not liking the old format and half of em liking it make the overall number more inclined towards change since the other 50% of the people did NOT like the old format??

I am messing up a little still between the two.

Thanks
 AthenaDalton
PowerScore Staff
  • PowerScore Staff
  • Posts: 296
  • Joined: May 02, 2017
|
#36108
Thanks for your question!

The answer choice that most supports the magazine's decision will be the one that results in the highest number of subscribers liking the new format.

The problem with making decisions about what all readers will think based on what a small number of survey respondents think is that the people who return the surveys may not be representative of all readers.

Imagine that 100,000 people subscribe to this magazine, and only 100 people return the surveys. Even if all 100 supported the change, would this really give us a good idea that ALL subscribers will like the new format? Maybe the only readers who bothered to respond were the ones that felt strongly about the change. Let's see how this works in practice --

In answer choice (C), we are told that the readers who returned the surveys (with 62% voting in favor of the new format) are almost exactly representative of the magazine's readership as a whole. So for answer choice (C) we know that very close to 62% of the readers will like the new format. This solves our survey sample size problem.

With answer choice (E) we learn that 90 percent of dissatisfied readers returned the survey and 50 percent of satisfied readers returned the questionnaires. The problem here is that we don't know what size either of the groups are.

Just as a hypothetical for answer choice (E), imagine that this magazine has 100,000 subscribers. Now let's assume that 100 readers hate the format (dissatisfied readers) and the remaining 99,900 readers love the format (satisfied readers). All of a sudden the fact that 90 percent of the really tiny dissatisfied group doesn't look as convincing, right? Here's how the actual math would shake out:

50 percent of 99,900 satisfied readers -- 49,950 satisfied readers responded
90 percent of 100 dissatisfied readers -- 90 dissatisfied readers responded

This is obviously more math than you would ever want to do on the LSAT. Just keep in mind that knowing that 90 percent of a group dislikes something doesn't give you any reliable information unless you know how that group's size compares to the whole.

Good luck studying!

Athena Dalton
 Lawyered
  • Posts: 23
  • Joined: Jun 13, 2017
|
#36145
Yes, I understand. I haven't fully gotten to the numbers and math part of the Bible yet but I have experienced similar questions and am starting to learn the trick. Just because there is a % given does not mean much unless there's numeric data to back it up.

I had the doubt of the % not being big enough or equally representative but figured I'd ask.

Thank you!!
 pklopfen
  • Posts: 4
  • Joined: Jul 19, 2021
|
#95397
I got this question right but am trying to foolproof my understanding of it. My largest problem is that I think the correct answer, while better than the others, is not actually correct per the question stem.

My problem lies within a couple of vague words:
- The stimulus states that the journal sent out a survey. It DOES NOT say whether that survey was sent to current readers, non-readers, or a mixture of both. If the survey was sent to non-readers then Answer C's note of SURVEYED READERS means that the 62% could be meaningless because we only have the percentage of those who supported the change that also returned the survey. Say the survey was sent to 100 people. 50 current readers and 50 non-current readers. Of those only the 50 non-readers returned the survey. That means of those only 31 were in support. If none of the current readers returned the survey maybe they all hate the proposed change. Then the journal could lose ALL of its current readership. Now, say the current readership is 1 million. Say the potential readership is only 20,000. If the journal loses even 3% of its current readership due to the change it wouldn't be able to make up for the lost readers even if it got 100% of the potential readers. Since the goal is to INCREASE readership it would be a net loss.

Basically I understand that this is a MOST STRENGTHEN and therefore the answer doesn't have to be airtight. It just has to be the best option out of the 5. It just seems a stretch from these numbers to conclude that from such a flimsy premise that the journal will INCREASE readership.
 Rachael Wilkenfeld
PowerScore Staff
  • PowerScore Staff
  • Posts: 1358
  • Joined: Dec 15, 2011
|
#95414
Hi pklopfen,

We aren't trying to prove the conclusion in a strengthen question. We are just trying to help it. Your question when you see an answer choice should be "does this make the conclusion more likely?" If yes, it strengthens. If not, it doesn't strengthen.

Here, we are increasing the value of the survey by stating that the survey is representative in a specific way. That's all. And that helps our conclusion because strengthening the underlying survey strengthens the conclusion. It makes it more likely to be true.

Strengthen answer choices can prove the conclusion, but they don't have to. Imagine a claim that says voting causes red hair. We could strengthen it by saying that John voted in the last election and John has red hair. Even though it's only one instance, it still helps the claim by making it slightly more likely to be true.

Hope that helps!
User avatar
 rrurrec
  • Posts: 3
  • Joined: Jun 17, 2022
|
#96004
Hi,

Wouldn't answer choice E actually weaken the question since it indicates that the survey was indeed a misrepresentation?

Get the most out of your LSAT Prep Plus subscription.

Analyze and track your performance with our Testing and Analytics Package.